LSAT and Law School Admissions Forum

Get expert LSAT preparation and law school admissions advice from PowerScore Test Preparation.

User avatar
 Dave Killoran
PowerScore Staff
  • PowerScore Staff
  • Posts: 5850
  • Joined: Mar 25, 2011
|
#44167
Complete Question Explanation
(The complete setup for this game can be found here: lsat/viewtopic.php?t=9437)

The correct answer choice is (C)

Answer choice (A) is incorrect because M and K cannot be selected together; alternately, if M is selected, than at least two Os must be selected.

Answer choice (B) is incorrect because K and O cannot be selected together.

Answer choice (C) is the correct answer.

Answer choice (D) is incorrect because if one O is selected, than at least two Os must be selected.

Answer choice (E) is incorrect because M and N cannot be selected together

Get the most out of your LSAT Prep Plus subscription.

Analyze and track your performance with our Testing and Analytics Package.